Ecologist: Without the intervention of conservationists, squirrel monkeys will become extinct. But they will survive...

Joseph on October 30, 2015

Why is D incorrect?

Looking for some guidance as to why D is incorrect and how to infer properly from the stimulus..

Reply
Create a free account to read and take part in forum discussions.

Already have an account? log in

Mehran on November 17, 2015

Thank you for your question. This is a "must be true" question, so we need to examine the stimulus carefully to determine what "must be true" according to its terms. I'm going to put the statements in a slightly different order so that we can follow them a bit more coherently.

First: Squirrel monkeys flourish in second-growth forest because of the plentiful supply of their favorite insects and fruit.

Second: Squirrel monkeys will become extinct without the intervention of conservationists.

Third: Squirrel monkeys will survive if large tracts of second-growth forest habitat are preserved for them.

All right. We know that the intervention of conservationists is necessary for squirrel monkeys' survival. We also know that if large tracts of second-growth forest are preserved for these monkeys, they will survive.

Now let's examine the answer choices.

There is no textual support for answer choice (A). Nothing in the stimulus establishes that the ONLY habitat that contains plentiful supplies of squirrel monkeys' favorite insects and fruit is second-growth forest.

Likewise, there is no textual support for answer choice (B). We don't know whether at least one of the conservationists who intervene to help the squirrel monkeys survive will do X, because we don't actually know that ANY conservationists WILL in fact intervene. We just know that such intervention is necessary for the monkeys' survival. That doesn't mean it will happen.

Answer choice (C) is a good example of LSAT trickery--they're trying to confuse two distinct issues mentioned in the stimulus, in the hope that careless readers will select this answer choice. But recall the stimulus--it did NOT say that squirrel monkeys require a plentiful supply of their favorite insects and fruit. Don't fall for the trap!

Answer choice (D) is a logical error--we were told in the stimulus that conservationists' intervention is necessary for the squirrel monkeys' survival, but we have NEVER been told that such intervention is sufficient to ensure that survival. You can't just reverse!

Answer choice (E) must be true. It combines the facts presented in the second and third parts of the stimulus as I have categorized them above.

Hope this helps! Please let us know if you have any additional questions.